The pregnancy length in days for a population of new mothers can be approximated by a normal distribution with a mean of days and a standard deviation of days. ​(a) What is the minimum pregnancy length that can be in the top ​% of pregnancy​ lengths? ​(b) What is the maximum pregnancy length that can be in the bottom ​% of pregnancy​ lengths? ​(a) The minimum pregnancy length is 280 days.

Answers

Answer 1

Answer:

(a) 283 days

(b) 248 days

Step-by-step explanation:

The complete question is:

The pregnancy length in days for a population of new mothers can be approximated by a normal distribution with a mean of 268 days and a standard deviation of 12 days. ​(a) What is the minimum pregnancy length that can be in the top 11​% of pregnancy​ lengths? ​(b) What is the maximum pregnancy length that can be in the bottom ​5% of pregnancy​ lengths?

Solution:

The random variable X can be defined as the pregnancy length in days.

Then, from the provided information [tex]X\sim N(\mu=268, \sigma^{2}=12^{2})[/tex].

(a)

The minimum pregnancy length that can be in the top 11​% of pregnancy​ lengths implies that:

P (X > x) = 0.11

⇒ P (Z > z) = 0.11

z = 1.23

Compute the value of x as follows:

[tex]z=\frac{x-\mu}{\sigma}\\\\1.23=\frac{x-268}{12}\\\\x=268+(12\times 1.23)\\\\x=282.76\\\\x\approx 283[/tex]

Thus, the minimum pregnancy length that can be in the top 11​% of pregnancy​ lengths is 283 days.

(b)

The maximum pregnancy length that can be in the bottom ​5% of pregnancy​ lengths implies that:

P (X < x) = 0.05

⇒ P (Z < z) = 0.05

z = -1.645

Compute the value of x as follows:

[tex]z=\frac{x-\mu}{\sigma}\\\\-1.645=\frac{x-268}{12}\\\\x=268-(12\times 1.645)\\\\x=248.26\\\\x\approx 248[/tex]

Thus, the maximum pregnancy length that can be in the bottom ​5% of pregnancy​ lengths is 248 days.


Related Questions

Name a real world context to describe the sums of rational numbers.

Answers

Step-by-step explanation:

when you are cooking you need to measure fractions of ingredients

need help please. Will give you 5-stars and a big thank you comrades

Answers

Answer:

first answer

Step-by-step explanation:

(8x³ - 22x² - 4) / (4x - 3)

when you do long division you get the first answer

Which of the following symbols could correctly finish the statement. Select all that apply. 0___-8 = ≠ > < ≥ ≤

Answers

Answer:

>

Step-by-step explanation:

Even though its 0 its still greater than any negative number.

Answer:

Step-by-step explanation:

Please help me understand this. Thank you! Gina has borrowed 100 songs from her friend. She plans to download an equal number of songs on her music player each week for 5 weeks. The graph shows the number of songs left to download, y, for a certain number of weeks, x: A graph titled Song Downloading shows the Number of Weeks on x-axis and Number of Songs Left to Download on the y-axis. The x-axis scale is shown from 0 to 5 at increments of 1, and the y-axis scale is shown from 0 to 140 at increments of 20. A straight line joins the ordered pairs 0, 100 and 1, 80 and 2, 60 and 3, 40 and 4, 20 and 5, 0. Part A: What is the rate of change and initial value of the function represented by the graph, and what do they represent in this scenario? Show your work to find the rate of change and initial value. (6 points) Part B: Write an equation in slope-intercept form to model the relationship between x and y. (4 points)

Answers

Answer:

Rate of Change/Slope = -20

Equation: y= -20x +100

Step-by-step explanation:

A. We know the rate of change is also known as the slope.  If we used the slope formula to find the slope we can find the Rate of Change.

[tex]\frac{y2 - y1}{x2 - x1} = \frac{100-80}{1-2} = \frac{20}{-1} = -20[/tex]

B. Since we know the slope and 1 point on the graph we can substitute them in for 'b'

(0,100)

(100)=-20(0) + b

b = 100

Since we know the slope and the b value we can write the equation:\

y = -20x +100

The rate of change refers to the slope of the line, that is the change in y-axis per unit change in the value on the x-axis. Hence, the rate of change is :

Slope = - 20y = -20x + 100

Slope = Rise / Run

Rise = (y2 - y1) = (0 - 100) = - 100

Run = (x2 - x1) = (5 - 0) = 5

Slope = - 100 / 5 = - 20

General form of a slope - intercept relation :

y = bx + c

The intercept, c can be calculated thus:

100 = - 20(0) + c

100 = 0 + c

c = 100

Hence, the slope - intercept equation will be y = - 20x + 100

Learn more : https://brainly.com/question/18479471

stagg high school has a rectangular swiming pool the area of the water in the pool is 1,800 meters squared the length is twice the width what is the perimeter of the pool find the length and width. SHOW WORK

Answers

Step-by-step explanation:

L*b=1800m^2

L=2b

2b*b=1800

2b^2=1800

b^2=900

b=30m

L=2*30

=60m

Perimeter=2(l+b)

=2(60+30)

=2*90

=180m

Show all work to solve the equation for x. If a solution is extraneous, be sure to identify it in your final answer.

square root of the quantity x minus 3 end quantity plus 5 equals x

Answers

Answer:

Step-by-step explanation:

[tex]\sqrt{x-3} +5=x\\\sqrt{x-3} =x-5\\squaring ~both~sides\\x-3=x^2-10x+25\\x^2-10x-x+25+3=0\\x^2-11x+28=0\\x^2-7x-4x+28=0\\x(x-7)-4(x-7)=0\\(x-7)(x-4)=0\\x=7,4[/tex]

put x=7 in the given equation

[tex]\sqrt{7-3} +5=7\\\sqrt{4} +5=7\\2+5=7\\7=7[/tex]

which is true .

∴ x=7 is a solution of the given eq.

now put x=4 in the given eq.

[tex]\sqrt{4-3} +5=7\\1+5=7\\6=7\\[/tex]

which is not true.

∴x=4 is an extraneous solution.

Complete the table. At least the first few so I understand how to do it

Answers

Answer:

What we need to do is simply multiply the values in both columns e.g 4 * 3/36 = 12/36

Please check explanation for complete answer

Step-by-step explanation:

Here, we are concerned about filling the empty columns of the table.

What we want to do here is simply straightforward. All we need to do is to

multiply the values of x by the values of P(x) in each of the individual rows.

Also recall, we do not need to reduce the fractions.

So we have;

2. 3 * 2/36 = 6/36

3. 4 * 3/36 = 12/36

4. 5 * 4/36 = 20/36

5. 6 * 5/36 = 30/36

6. 7 * 6/36 = 42/36

7. 8 * 5/36 = 40/36

8. 9 * 4/36 = 36/36

9. 10 * 3/36 = 30/36

10. 11 * 2/36 = 22/36

11. 12 * 1/36 = 12/36

help8b2 • 2b3 a) 16b-2 ,b) 16b6 ,c) 16b-4 ,d) 16b5

Answers

Answer:

16b^5

Step-by-step explanation:

8b^2×2b^3

= 16b^2+3

= 16b^5

Answer:

D)16b5

Step-by-step explanation:

8b2×2b3

= 16b2+3

= 16b5

Find f(x) and g(x) so the function can be expressed as y = f(g(x)). (1 point) [tex]y=\frac{7}{x^{2} } +10[/tex]

Answers

Answer:

The functions are [tex]f(x) = 7\cdot x+10[/tex] and [tex]g(x) = \frac{1}{x^{2}}[/tex], respectively.

Step-by-step explanation:

Let suppose that [tex]g(x) = \frac{1}{x^{2}}[/tex], then [tex]f(g(x))[/tex] is:

[tex]f(g(x)) = 7\cdot \left(\frac{1}{x^{2}} \right) + 10[/tex]

[tex]f(g(x)) = 7\cdot g(x) + 10[/tex]

Thus,

[tex]f(x) = 7\cdot x + 10[/tex]

The functions are [tex]f(x) = 7\cdot x+10[/tex] and [tex]g(x) = \frac{1}{x^{2}}[/tex], respectively.

HELP ASAP ITS SO HARD! Kelsey did the following division problem. Her teacher says that the quotient she found is wrong. −2 5/6 ÷ 1 1/3 −17/6 ÷ 4/3 −6/17• 3/4 −6×3 divided by 17×4 −18/68 −9/34 A. Identify what Kelsey did wrong in her calculations. B. Find the correct quotient, showing all of your calculations.

Answers

Part A

Her steps were

[tex]-2 \frac{5}{6} \div 1 \frac{1}{3}\\\\-\frac{17}{6} \div \frac{4}{3}\\\\-\frac{6}{17} \times \frac{3}{4}\\\\-\frac{6\times 3}{17\times4}\\\\-\frac{18}{68}\\\\-\frac{9}{34}\\\\[/tex]

Kelsey made a mistake on line 3. Note how the 17/6 flips to 6/17. This is not correct. You keep the first fraction the same, but you do flip the second fraction. This only applies when you divide two fractions.

The third step should look like [tex]-\frac{17}{6}\times \frac{3}{4}[/tex]

=======================================================

Part B

Here's what she should have written

[tex]-2 \frac{5}{6} \div 1 \frac{1}{3}\\\\-\frac{17}{6} \div \frac{4}{3}\\\\-\frac{17}{6} \times \frac{3}{4}\\\\-\frac{17\times 3}{6\times 4}\\\\-\frac{51}{24}\\\\-\frac{17}{8}\\\\[/tex]

If you want to convert that improper fraction to a mixed number, then you could do something like this

[tex]-\frac{17}{8} = -\frac{16+1}{8}\\\\-\frac{17}{8} = -\frac{16}{8}-\frac{1}{8}\\\\-\frac{17}{8} = -2 \frac{1}{8}\\\\[/tex]

Or you could divide 17 over 8 using long division to get 2 remainder 1. The 2 is the quotient that goes to the left of the 1/8. The remainder of 1 is the numerator of 1/8.

a hotel manager wants miriam to tile their lobby using the dame design she created for Mr.Rivera.The lobby measures 45 feet by 45 feet. he wants the outer edge to be the same color as the center tile. will this occur ? justify your answer

Answers

Answer:

Yes it will occur

Step-by-step explanation:

The lobby measures 45 feet by 45 feet

Area of the lobby = 45 * 45

=2025 ft^2

So, the lobby has 2025 tiles

subtract 1 black tile in the center

2025 tiles - 1 black tile =2024 tiles

The number of blue tiles and black tiles is 2024 tiles

He wants the outer edge to be the same color as the center tile so, divide by 2

2024/2 = 1012 tiles

The number of tiles in the outer edge is 1012 tiles and the number of tiles in the center is 1012 tiles

Wait times at a dentist's office are typically 21 minutes, with a standard deviation of 2 minutes. What percentage of people should be seen by the doctor between 17 and 25 minutes for this to be considered a normal distribution?

Answers

ANSWER: 95%

HOW:
95% of a group of data with a normal distribution is between two standard deviations to the right and left

Answer:

95%

Step by step explanation:

z = 17-21 / 2 and z = 25-21/2

z=-2 (2.28%) z=2 (97.72%)

97.72 - 2.28 = 5.44

100% - 5.44% is about equal to 95%

what is the answer for 6x-4=-26+5x

Answers

Answer: x=-22

Step-by-step explanation:

    6x-4=-26+5x

6x-4-5x=-26+5x-5x ⇔ subtraction property of equality

       x-4=-26

  x-4+4=-26+4 ⇔ addition property of equality

         x=-22

Answer:

x = - 22

Step-by-step explanation:

6x - 4 = - 26 + 5x

First of all group like terms

Send the constants to the right side of the equation and those with variables to the left

That's

6x - 5x = 4 - 26

Simplify

We have the final answer as

x = - 22

Hope this helps you

a second degree equation in one variable example how many solutions does it have ?a second degree equation in one variable example how many solutions does it have ? is it possible to have many solutions or no solutions tions give an example for each

Answers

Answer:

  0, 1, or 2 real solutions

Step-by-step explanation:

Including complex and repeated solutions, a polynomial with real coefficients, and of degree n, always has n solutions.

If you're only concerned about real solutions, a 2nd degree equation in one variable may have 0, 1, or 2 real solutions. Here are some examples.

0 solutions: x^2 +1 = 01 solution: x^2 = 02 solutions: x^2 -1 = 0

PLEASE HELP Ruri is a 30-year-old math teacher. She has been informed that she is the winner of a grand prize for the lottery. She can choose either a one-time payment of $20 million or $5000 per week for the rest of her life. Which choice would most likely result in the greatest amount of winnings for Ruri? Explain your reasoning.

Answers

Answer:

$5,000 per week

Step-by-step explanation:

Ruri is a 30 year old female.

there are about 4 weeks per month

there are about 52 weeks per year

52*5000 = 260,000

She would get 260,000 per year and lets see how much she would have at 40.

260,000*10

at 40 she would have 2,600,000

2,600,000*10

at 50 she would have 26,000,000

at 50 she already has earned more money that the $20 million.

She should go with the $5000 per week if she would like more money.

If the geometric mean of a and 120 is 60, find the value of a.

Answers

Answer:

The answer is option 4.

Step-by-step explanation:

Given that the mean formula is total score/number of score. So we can assume that total score is (a+120), number of score is 2 and the mean value is 60. Then you have to find the value of a :

[tex] \frac{a + 120}{2} = 60[/tex]

[tex]a + 120 = 60 \times 2[/tex]

[tex]a + 120 = 120[/tex]

[tex]a = 120 - 120[/tex]

[tex]a = 0[/tex]

[tex] \text{ Geometric mean of two numbers } a \text{ and } b , \text{ where } a,b>0 \text{ is given by } G= \sqrt{ab}[/tex]

[tex] \sqrt{a\cdot120}=60 [/tex]

[tex] \implies 60 \cdot 2 \cdot a=60\cdot 60 \qquad \text{Squaring both sides} [/tex]

[tex] \implies a =30 [/tex]

what equation accurately represent this statement three less than 4 times a number is less than 12

Answers

Answer: 4t - 3 < 12

Step-by-step explanation:

Two students use different methods to solve this multiplication problem:
3/4*-4 2/9
Read each of their methods below and then enter numbers to correctly complete their
work.

Answers

Answer/Step-by-step Explanation:

Given, [tex]\frac{3}{4}*-4\frac{2}{9}[/tex]

Ivy solves this problem by writing each number as a fraction and then multiplies as shown below:

[tex] \frac{3}{4}*-4\frac{2}{9} = \frac{3}{4}* -\frac{38}{9} [/tex]

[tex] = \frac{3}{4}*-\frac{38}{9} = -\frac{3*38}{4*9} = -\frac{1*19}{2*3}[/tex]

[tex]= -\frac{19}{6}[/tex]

Fabian solves this problem by writing the mixed number as a sum and applies the distributive method of multiplication as shown below:

[tex] \frac{3}{4}*-4\frac{2}{9} = \frac{3}{4}(-4 + -\frac{2}{9}) [/tex]

[tex] = \frac{3}{4}*-4 + \frac{3}{4}*-\frac{2}{9} [/tex]

[tex] = -\frac{3}{1} + \frac{1}{2}*-\frac{1}{3} [/tex]

[tex] = -3 + (-\frac{1}{6}) [/tex]

[tex] = -3 - \frac{1}{6} = -3\frac{1}{6} [/tex]

help help help me plZZZZZ ill give you brainly ;DDD

Answers

Answer:

the answer is 60.7

Step-by-step explanation:

60 to has a between numbers like given in the picture

so as number line it's

60.1 . 60.2 60.3 60.4 60.5 60.6 60.7 60.8 and continue

if u get any 3 digit number like 600 to 650 in number line

u do it like it the same 600.1 600.2.... and go on

Answer:

63½ or 63.5

Step-by-step explanation:

65-60=5

10points=5

1point=?

1×5/10= ½

that means the sequence continues after adding ½ i.e

60..60½...61...61½...62...62½...63...63½...64..64½...65

you have been asked the 8th number which is 63½

help plz I think the first one is correct but I'm not sure

Answers

Answer: You are correct.

The left side 10x+25 is the cost expression for Black Diamond, while the right hand side 5x+50 is for Bunny Hill. The x is the number of hours.

how to find the theta with side lengths of a triangle

Answers

Step-by-step explanation:

Hello, there!!!

I hope you mean the question is like the above problem in picture.

so, let's simply work with it.

here, we may use cosine rule,

so, according to cosine rule,

[tex] {c}^{2} = {a}^{2} + {b}^{2} - 2ab.cosc[/tex]

so, just put value of formulae here,

we get;

5^2 = 3^2 + 4^2 - (2×3×4) . cos thita

or, 25 = 9 + 16 -24 cos thita.

or, 24 cos thita = 0

or, cos thita = 0/25

or, cos thita = 0

now, taking cos to right side we get,

[tex] {cos}^{ - 1 } (0)[/tex]

now, after typing cos ^-1 (0) we get angle as 90°.

(note: in step {cos thita = 0} you couold directly write like; cos thita = cos 90°. and cos would be cancelled in it as cos 90°=0. but it is only applied in particular angle like 0°,30°,60°,..... which are identified or if you don't know you must use the method above using calculator and remember to put inverse {cos^-1}).

so, In this way we find angle.

I hope it helps....

help me im dangered plzzzzzzzzzzzzzzzzzzzz

Answers

Answer:

A

Step-by-step explanation:

Hi!

An exponent is the same thing as just multiplying the expression by itself the number of times the exponent says. So we need to multiply 1/3 by itself three times.

1/3 * 1/3 * 1/3 = 1/27

the answer to your question is A

What is the y−intercept of the line that passes through the point (4,9)and is parallel to the line y=12x+2?

Answers

Answer:

y- intercept = - 39

Step-by-step explanation:

The equation of a line in slope- intercept form is

y = mx + c ( m is the slope and c the y- intercept )

y = 12x + 2 ← is in slope- intercept form

with slope m = 12

Parallel lines have equal slopes, thus

y = mx + c ← is the partial equation

To find c substitute (4, 9) into the partial equation

9 = 48 + c ⇒ c = 9 - 48 = - 39 ← y- intercept

Answer:

y-intercept = -39

Step-by-step explanation:

if two lines are parallel it means they have the same gradient so we compare the equation given to the default equation of a line

y=mx+c

y=12x+2

comparing we have the gradient m=12 now finding the equation of the line parallel to the given line we use

y-y1=m(x-x1)

y1=9 and x1=4

y-9=12(x-4)

y-9=12x-48

y=2x-48+9

y=2x-39

comparing to the default equation of a line y=mx+c where c is the y-intercept

therefore the y-intercept is -39

Please answer this question now

Answers

Answer:

30.9 cm²

Step-by-step explanation:

To find the surface area of this figure, we find the area of the base and the 3 identical sides.

The base is split into two identical right triangles. Let's find the area of one and multiply by two.

Half of 3: 1.5

[tex]1.5\cdot2.6=3.9\\3.9\div2=1.95[/tex]

There are two right triangles:

[tex]1.95\cdot2=3.9[/tex]

The area of one of the sides will be the same thing, except the height is 6.

[tex]1.5\cdot6=9\\9\div2=4.5\\4.5\cdot2=9[/tex]

There are 3 sides identical to this one:

[tex]9\cdot3=27[/tex].

Add 27 and 3.9:

[tex]27+3.9=30.9[/tex]

Hope this helped!

Answer:

30.9 square centimeters

Step-by-step explanation:

3 * 1/2(3)(6) + 1/2(3)(2.6) = 30.9

Which data set matches the box-and-whisker plot?
A) 12 13 15 19 23 23 25 26.5 28 30
B) 15 13 19 21 23 24 27 29 32
C) 11 31 13 15 19 21 21 25 27 29 31
D) 11 13 15 19 23 23 24 26.5 28 33​

Answers

Answer:

D) 11 13 15 19 23 23 24 26.5 28 33​

Step-by-step explanation:

The box-and-whisker plot displayed above has the following key values that we can use to identify which of the given data set it matches. It has:

Minimum value = 11

Q1 = 15

Median = 23

Q3 = 26

Maximum value = 33

From the options given, using just the max and min value, we can conclude that the data set in option D matches the box plot.

The data set in option D has a minimum value of 11, and a maximum value of 33.

ASAP PLZ ANSWER!!! Can you tell me step by step to this question 8,595 ÷ 24?

Answers

Answer:

358 and remainder of 3

Step-by-step explanation:

1. Divide it like any other problem

24 goes into 85, 3 times with 13 left overBring down the 9 and 24 goes into 139, 5 times with 19 left overThen bring down the 5 and 24 goes inside 195, 8 times with 3 left overSo your remainder would be 3

Hope this helps

simplify 3/7
into a whole number

Answers

Answer:

You can't because it's not a "full fraction" like 7/7. The best you can do is turn that into a decimal like 0.43

Step-by-step explanation:

-7p+2(5p-8)=6(p+6)-7

Answers

Answer:

-15

Step-by-step explanation:

-7p+10p-16=6p+36-7

3p-16=6p+29

3p-6p=29+16

-3p=45

p=45/-3

p=-15

P=5

-7p+2(5p-8)=6(p+6)-7
Distribute
-7p+10p-16=6p+36-7
Add -7p and 10p together
3p-16=6p+36-7
Subtract 7 from 36
3p-16=6p+29
+6p (on both sides)
9p-16=29
+16 (on both sides)
9p=45
Divide both sides by 9
P=5

**Yoxelt buys 4 1/2 gallons of soda. One-fourth of the soda he bought was Pepsi and the rest was Sprite. How many gallons of Pepsi did Yoxelt buy? Show all work below.

Answers

Answer:

  1 1/8

Step-by-step explanation:

1/4 of the 4 1/2 gallons were Pepsi, so the amount is ...

  (1/4)(9/2) = (1·9)/(4·2) = 9/8 = 1 1/8

Yoxelt bought 1 1/8 gallons of Pepsi.

Which of the following are natural numbers? There may be more than one correct answer. Select all that apply. If only one answer is correct, select "only" and the answer that applies. A.) only B.) −1,−2,−3,… C.) 7,8,9,… D.) fractions E.) 22

Answers

Answer:

Option C and option E

Step-by-step explanation:

C.) 7,8,9,…

E.) 22

Natural numbers are also called counting numbers.

They begin from 1, 2, 3 to infinity.

Natural numbers are greater than zero (0)

They do not have decimal point in them.

They are positive integers, as such they do not have minus

Natural numbers can include commas when they are large like 3,000

Other Questions
Theresa bought 2 pineapples for $6. She be wants to find the constant of proportionality in terms of dollars per pineapple. She modeled this proportional relationship on a number line diagram, as shown. Part AUsing the diagram, find the constant of proportionality in terms of dollars per pineapple. Read this excerpt from "The Bells by Edgar Allan Poe. In the startled ear of night How they scream out their affright! Too much horrified to speak, They can only shriek, shriek, Out of tune, Which statement best describes how Poe creates mood in the excerpt? Poe uses onomatopoeia to create a frustrated mood. Poe uses onomatopoeia to create a serious mood. Poe uses onomatopoeia and rhyme to create a mood of panic. Poe uses onomatopoeia and rhyme to create a mood of disbelief. solve for x 3(x+2) = 12 A box is 1 m high, 2.5 m long, and 1.5 m wide, what is its volume? A lab technician is dividing a cell that has a diameter of 4.32104 4 . 32 10 - 4 millimeters. Each of the new cells has a diameter measuring exactly one half of the diameter of the original cell. Which is the diameter of a new cel Oriole Company purchased equipment for $41600. Sales tax on the purchase was $2496. Other costs incurred were freight charges of $624, repairs of $364 for damage during installation, and installation costs of $696. What is the cost of the equipment Which statement BEST describes an organ? of a portfolio. The beta of four stocksG, H, I, and Jare , , , and , respectively. What is the beta of a portfolio with the following weights in each asset: LOADING...? What is the beta of portfolio 1? The market for plywood is characterized by the following demand and supply equations: QD = 800 10P and QS = 50P 1,000, where P is the price per sheet of plywood and Q measures the quantity of plywood. What is the size of the deadweight loss if the government imposes a price ceiling of $25 per she Choose three organizations that you believe have had the greatest impact on the current state of safety and health programs in the United States. Summarize the purpose of each organization, and discuss how you believe you can use these organizations to improve your ability to perform your duties as a safety and health professional. MATH QUESTION : As punishment for bad behavior during recess, Mrs. Busywork asked her class to multiply 10 by 1/3 five times. John, however, notices that it is possible to multiply 10 by a single fraction and still get the same answer as the other students. What is this single fraction? An investor holds a 10 year bond pays a coupon rate of 9%. The yeid to maturity of the bond is 10% . The bond is trading: which of the following best explains why it is important to protect rivers? a) without them, animals in marine ecosystems would become overpopulated. b) we depend on them to meet our basic needs by providing drinking water. c) without them, we would lose money made through overseas trade. d) we depend on them to provide us with our main source of food. A result of a number, what decreased by 15% equal 161. Find that number. hich of the following cases would the U.S. Supreme Court be the first to hear? a civil trial between residents of one U.S. state a civil trial where neighbors sue one another a criminal trial involving a U.S. ambassador a criminal trial for a suspect in a murder Which is a likely result of taking LSD? A. raised heart rate and sped-up nervous system B. altered mood and sensory perceptions C. reduced heart rate and loss of inhibitions' Which event marks the beginning of the "rising action" in "The Most Dangerous Game"?Rainsford's dinner with ZaroffRainsford trapping ZaroffRainsford falling off the yachtRainsford being hunted in the figure above, pqrs is a parallelogram. What is the value of x? Explain dowry murder HCF of x minus 2 and X square + X - 6